LSAT and Law School Admissions Forum

Get expert LSAT preparation and law school admissions advice from PowerScore Test Preparation.

User avatar
 Dave Killoran
PowerScore Staff
  • PowerScore Staff
  • Posts: 5852
  • Joined: Mar 25, 2011
|
#80604
Complete Question Explanation
(The complete setup for this game can be found here: lsat/viewtopic.php?t=8622)

The correct answer choice is (D)

The inference involving the remaining jail was discussed at the end of the setup examination. The remaining jail cannot be placed in cities 2, 3, and 6, and only cities 1, 4, and 5 remain as candidates. Thus, answer choice (D) is correct.

Get the most out of your LSAT Prep Plus subscription.

Analyze and track your performance with our Testing and Analytics Package.